LSAT and Law School Admissions Forum

Get expert LSAT preparation and law school admissions advice from PowerScore Test Preparation.

 Administrator
PowerScore Staff
  • PowerScore Staff
  • Posts: 8916
  • Joined: Feb 02, 2011
|
#28610
Complete Question Explanation
(The complete setup for this game can be found here: lsat/viewtopic.php?t=11942)

The correct answer choice is (D)

The condition in the question stem indicates that M is delivered fourth. For M to be delivered fourth, only H, J, and N can be delivered before M (all three must be delivered before M regardless, but to allow M to be delivered fourth those can be the only three parcels delivered before M). A diagram including the new condition would appear as:
Dec 06_M12 game #4_cr_game#4_#18_diagram_1.png
The arrows “bracketing” K indicate that K is delivered after M but before O. K has no relationship with G or L. Other than the placement of K, the diagram is relatively standard.

From a linear standpoint, this creates the following scenario:
Dec 06_M12 game #4_cr_game#4_#18_diagram_2.png
Use the above information to attack the answer choices.

Answer choice (A): This answer choice is incorrect because G could be delivered fifth, sixth or seventh.

Answer choice (B): This answer choice is incorrect because O could be the eighth parcel delivered.

Answer choice (C): This answer choice is incorrect because the relationship between J and H is unfixed, and thus H could be delivered later than J.

Answer choice (D): This is the correct answer choice. N must always be one of the first three parcels delivered and K must be delivered fifth, sixth, or seventh.

Answer choice (E): This answer choice is incorrect because G and L do not have a fixed relationship, and therefore L could be delivered later than G.
You do not have the required permissions to view the files attached to this post.

Get the most out of your LSAT Prep Plus subscription.

Analyze and track your performance with our Testing and Analytics Package.